sábado, 12 de maio de 2018

No mito a verdade é revelada pelos deuses; na metafísica ela deve ser buscada, achada com o recurso da razão, com o esforço do ser humano. Portanto, se o mito é criação do ser racional, logo, os deuses que revelam verdades através do mito são seres humanos?

Não. Os deuses são abstrações, figuras imaginárias inventadas pelo ser humano. Essas invenções, assim, são entidades que que possuem capacidades que o ser humano gostaria de possuir, como o poder de agir sobre a natureza a seu bel prazer e não segundo as regras dela. Por observar que, em grande parte dos casos, as ocorrências podem ser atribuídas a seres palpáveis dotados de escolha, decisão e poder de ação sobre a natureza, os homens primitivos, por analogia, consideravam que todas as ações, como tempestades, doenças, morte, catástrofes em geral e várias outras, também seriam resultantes de uma escolha, uma decisão e uma ação de um agente consciente super poderoso, mas invisível, que, então, eles corporificavam como um deus, inclusive, dotando-os de aspecto antropomórfico, muitas vezes mesclado com o de outros animais tidos como poderosos. São raras as concepções de deuses como entidades amórficas. Por outro lado, as proposições que esses deuses inventados exaram, ora como explicações, ora como prescrições, são as que as os líderes das comunidades gostariam que elas considerassem que fossem verdades, o que passa a ser altamente plausível se consideradas como pronunciadas pelas ditas divindades. Em verdade os mitos não têm nenhuma intenção de buscar a verdade, mas apenas de propiciar uma teoria com certa consistência e aceitabilidade (isto é, não absurda, dentro do que possa ser admitido) que apazigue a curiosidade das pessoas e, ao mesmo tempo, as incite em agir em consonância com o que os detentores do poder assim desejem.

Em algum momento da vida precisou enfrentar depressão ou algum problema como, ansiedade, professor? Qual é a sua dica para quem a tem a mais de 10 anos?

Às vezes eu fico triste, angustiado, ansioso ou tenso. Mas é algo temporário que se esvai naturalmente, não permanecendo como um quadro crônico. Depressão mesmo eu não sei o que seja. Mas costumo me sentir aborrecido e angustiado com o mundo como ele é, que é muito diferente do que como eu queria que fosse. Todavia, como sou uma pessoa desligada dos problemas de convivência em geral, isso não me deixa deprimido. O que costuma acontecer é eu ficar muito bravo quando presencio uma injustiça ou uma desonestidade. Então eu "solto os cachorros" seja em cima de quem for. Mas depois minha raiva passa. Nunca cheguei a odiar pessoa nenhuma. Nem mesmo "ficar de mau" com ninguém.

Qual é a tendencia na área de trabalho hoje em dia? As áreas em ascensão de expansão?

Não tenho a menor ideia. Nunca me preocupei com isso na vida. Sempre escolhi trabalhar naquilo que me dava gosto sem me preocupar com o rendimento. Na juventude eu pensava em ser engenheiro mecânico ou arquiteto, mas não pela renda que daria e sim porque eu gostava do assunto. Depois mudei para a Física e a Matemática por serem mais abstratas e filosóficas o que me agrada mais. E o magistério é uma área pela qual eu sou completamente fascinado. Eu pagaria para ser professor. Adoro lecionar. Felizmente nunca precisei procurar trabalho. Sempre fui convidado e sempre pelas melhores escolas. Então nunca cuidei de saber que área de trabalho é a melhor em termos de retorno econômico. Para mim a melhor é aquela que você gosta de atuar. Mas acho que o melhor é o comércio e a indústria. Não como empregado, mas como dono.

Universidade de Oxford é uma opção para se doutorar em física? Falo isso pois meu ex professor de experimental 1 obteve seu doutorado lá e mostrou uma didática incrível a nós, bem como um conhecimento sem par da área que ele se especializou, que foi física de materiais, física da matéria condensada.

Sim, uma das melhores. Mas a didática do seu professor é um dom pessoal dele. Não se aprende em nenhuma doutorado.
https://www.topuniversities.com/university-rankings/university-subject-rankings/2016/physics-astronomy
http://www.thebestschools.org/rankings/best-physics-programs-in-world-today/

Pessoas de classes sociais mais elevadas tendem a ser mais egoístas ou não?

Não tenho levantamentos estatísticos a respeito desse fato. Minha impressão, "a sentimento", e portanto sem validade científica, é de que sim, pessoas economicamente mais privilegiadas tendem a ser mais egoístas. Mas nem tanto as pessoas socialmente mais elevadas que não sejam economicamente mais privilegiadas. Isso não significa que não existam pobres egoístas. É uma questão estatística de proporções dentro das categorias.

Comente a frase: "Ser pobre não é vergonha, mas também não é orgulho."

Não há o que comentar. É a verdade.

Muitos religiosos dizem querer ajudar as pessoas por esperar algo em troca de Deus, isso seria uma ajuda genuína ou apenas uma esperança de receber uma recompensa divina?

Do ponto de vista de quem recebe a ajuda não importa: a ajuda foi dada. Do ponto de vista de quem dá a ajuda, fazê-lo para merecer alguma recompensa, como a salvação eterna ou evitar a danação eterna, compromete o caráter ético da ação, que deixa de ser algo nobre e generoso e passa a ser algo interesseiro.

A eficiência da medicina tem prolongado indiscriminadamente a vida das pessoas, contrariando aparentemente o objetivo da seleção natural. Esta interferência ou resultado, a longo prazo pode ser considerado "um tiro no pé"?

Não. Apenas mostra que, para a espécie humana, a evolução por seleção natural não funciona mais. Nesse caso ela terá que ser feita artificialmente, pela própria Medicina, junto com a Biologia. Mesmo para animais domesticados, a evolução por seleção natural não mais funciona. A Zootecnia já promove a evolução desses animais.

Você tem experiência com olimpíadas de física? Quero ouro na OBF desse ano e preciso aprofundar alguns conteúdos por livros universitários. Preciso aprofundar em ondulatória, termologia, óptica e relatividade restrita. Estou entre Sears, Tipler e Serway, quias destes vc recomenda?

** Na última pergunta eu pedi a indicação de um dos livros no plural, foi sem querer. Quero apenas saber o melhor daqueles, que inclui a didática também. Não terei tempo para estudar mais de um dos livros/coleções. Héllio
Não conheço o Serway. Entre o Sears e o Tipler, prefiro o Sears (o mais moderno). Mas os assuntos que você abordou não estão todos no mesmo volume.

Ernesto, meu professor de eletromagnetismo quer que a gente faça um trabalho mostrando a dedução da fórmula pra calcular o campo el. De um anel carregado em um ponto. Ele quer isso sem plágio, mas como vou deduzir sem seguir os mesmos passos? Seria para seguir a matemática mas com nossas palavras?

Faça um esquema e vá deduzindo as expressões por sua conta que, certamente, não sairá igual a nenhuma outra contida em livro algum. Mas não faça no eixo do anel, que é uma trivialidade. Faça fora do eixo do anel. Mas pode supor o anel uniformemente carregado. Esse é um trabalho realmente difícil. Como, por exemplo, o campo de um dipolo elétrico com solução exata em qualquer ponto fora do plano mediatriz do segmento que une as cargas. Faça em coordenadas polares (tanto um quanto o outro).

Professor, o Tomoshyio diz, ainda hoje, que a Mecânica Quântica é determinista. E você diz o contrário. Ambos são físicos, mas por quê tal divergência entre vocês, sendo que a física é um ramo que contém corte epistemológico?

Acontece que as Interpretações da Mecânica Quântica (mas não a teoria preditiva dela) ainda não atingiram o estágio de "corte epistemológico", havendo propostas divergentes ainda em aberto. Veja uma listagem nesses artigos:
https://pt.wikipedia.org/wiki/Interpreta%C3%A7%C3%B5es_da_mec%C3%A2nica_qu%C3%A2ntica
https://en.wikipedia.org/wiki/Interpretations_of_quantum_mechanics
http://materias.df.uba.ar/f4Aa2012c2/files/2012/08/mq2.pdf
https://books.google.com.br/books?id=IHUdnQEACAAJ&redir_esc=y
https://books.google.com.br/books?id=f1yqHAAACAAJ&redir_esc=y

Fui uma criança e adolescente bastante tímida e sofria com isto. Não sei de que modo aquela timidez foi embora, mas tenho a certeza que muitos sofrem, ainda hoje, com isto. Como acabar com a timidez?

Não sei, porque nunca fui tímido, mesmo sendo introspectivo em muitas ocasiões, mas nem sempre. Acho que só um psicólogo poderá responder. Não vejo problema nenhum em falar em público ou em me dirigir a uma pessoa tida como importante. Talvez porque tenha aprendido com meu pai que ninguém é mais nem menos importante do que eu mesmo. Para mim todo mundo é igual. Não sei o que leva alguém a ter esse receio da aprovação dos outros que chega até a paralisar a pessoa. Para mim se alguém não me aprovar, azar. Não estou nem aí. Faço e digo o que eu quero, seja para quem for. Se isso me prejudicar, então fico prejudicado. O que tem de mais? Não ligo para levar bronca e nem prejuízo. Não ligo para ser repreendido. E repreendo quem eu vir que merece, não importa quem seja. Nem o juiz nem o delegado. Aprendi com minha mãe, que nunca pensava duas vezes para dizer o que lhe ia na cabeça, "na lata". Meu pai também era assim. E o interessante é que eles eram pessoas muito respeitadas porque eram muito verdadeiros e francos. Nunca eram bajuladores de ninguém, como eu não sou. Tinham consideração para com todo mundo. Mas não tinham consideração nenhuma para mentirosos, enganadores, malvados, trapaceiros, injustos, aproveitadores e essa laia de gente. Especialmente políticos, empresários, padres e pastores. Minha mãe contestava os padres de frente, perante todo mundo. Eu adorava meu pai e minha mãe por serem assim e os imitei. Mas como eles trabalhavam pra valer e faziam muita caridade de verdade, metendo a mão na massa ao pé da letra, isto é, limpando o cocô da bunda da meninada de fato e não metaforicamente. Minha mãe parecia uma madre Tereza de Calcutá. Só que ela fazia uma caridade humanística, sem vincular a nada religioso. Mesmo ela sendo católica, ela achava que quem fazia a caridade não era Jesus, eram as pessoas. Então, quanto a essa questão da timidez, não sei responder. Esse tipo de problema nunca se apresentou para mim.

Ernesto, você é ou já foi pesquisador?

Logo que eu terminei meu mestrado o CNPQ ainda admitia mestres como pesquisadores, de modo que eu cheguei a fazer pesquisa. Logo depois passou a exigir o doutorado, que eu não tenho. Então eu ainda entrei em algumas pesquisas, mas não como líder. Depois eu assumi vários cargos administrativos na Universidade que não me deixavam tempo para pesquisar (chefe do departamento de física, coordenador do curso de física, pró-reitor de graduação, assessor do reitor e chefe de gabinete do reitor). Só continuei a dar aulas no Bacharelado de Física. Em verdade eu só cheguei a publicar a pesquisa derivada de minha tese de mestrado:

Se existisse inferno, você sentiria dor no inferno?

Há uma série de considerações. Para se considerar a existência de inferno, há que se considerar a existência de uma alma que saia do corpo na morte e tenha uma espécie de vida não biológica, como um espírito, que viveria, nesse caso, no inferno. Se espírito não é uma entidade física, não tem propriedades físicas, como volume e localização. Portanto o inferno não poderia ser um lugar, pois "lugar" é um volume de espaço com certa localização. O que seria, então? Por outro lado, dor é um processo biológico que se dá no sistema nervoso. Como o espírito não é biológico, não há como ele sentir dor, uma vez que não tem nervos, medula, cérebro etc. Nem há como ele ter sensações e nem pensamentos, já que esses processos também são ocorrências do sistema nervoso. Portanto espíritos, se existirem, não estão em lugar nenhum, não ocupam espaço nenhum e nem têm sensação nenhuma. Em suma, não vejo como considerar que existam. Assim, também não vejo como considerar que possa existir inferno, demônios, anjos e deuses.

Fazer justiça com as próprias mãos, pra você é certo ou não?

É errado. Mas a população pode prender o malfeitor e chamar a polícia para entregá-lo a ela, bem como entrar com ação popular contra ele. Todavia acho muito importante que a legislação se torne MUITO, mas MUITO mesmo, mais rigorosa em suas punições. Menos a pena de morte. Para isso é preciso que as pessoas se candidatem a cargos legislativos e votem em candidatos que façam essas propostas. Menos no Bolsonaro ou em candidatos de direita. Muitos pensam que um rigor punitivo é uma atitude direitista, mas não é. A verdadeira esquerda também é contra todo tipo de crime e defende um maior rigor nas punições. Claro que esse rigor TEM que ser acompanhado de outras medidas, como a elevação da renda da população, do nível de instrução, das oportunidades de trabalho (não necessariamente de emprego). Mas não é desculpa aceitar punições brandas porque essas outras medidas ainda não foram tomadas. E tem mais. As punições têm que ser tanto mais rigorosas quanto mais alto for o nível econômico e educacional do criminoso,. No caso do criminoso ser um político, um magistrado, um militar (ou policial), um sacerdote, um professor, então é que as punições têm que ser mais rigorosas ainda. E é preciso acabar com as figuras da fiança, do sursis e de qualquer tipo de benefício, como prisão especial para portadores de curso superior, foro privilegiado para altos mandatários políticos e jurídicos, concessões de indultos e tudo isso. Quem tiver bom comportamento, que cumpra a pena toda. Quem tiver mau comportamento que a tenha estendida. Agora, é preciso que as penitenciárias sejam decentes e com número de vagas suficiente para a quantidade de detentos com sobra e acomodações humanas, sem conforto, certamente. Outra coisa é que todo presidiário tem que trabalhar para seu sustento na prisão. E em trabalhos braçais, seja o preso quem for. Outra coisa, ainda, é que o tempo de desenvolvimentos dos processos judiciais que levem à decisão final de condenação ou absolvição precisa ser reduzido drasticamente. Drasticamente mesmo. E ainda, que os condenados, desde a primeira instância, já sejam aprisionados. Se houver absolvição em alguma instância superior (que tem que ser, no máximo, de duas), então o estado indenizará o preso.

Ernesto, sei da sua ideologia de não juntar dinheiro e bens materiais, mas se não podemos, pelo menos por hora, ficarmos independentes do dinheiro, porque não poupar um pouco para, numa emergência, termos o dinheiro disponível ?

Pode fazer isso. Não tem problema. Eu não faço porque seria contraditório em relação a minhas convicções. De modo que, em alguma emergência, eu fico sem resolver o problema e pronto. Se eu perder o que tenho e ficar na miséria, não vejo problema nenhum. Miseráveis também vivem.

Corrija-me se eu estiver errado. Toda religião é mito. Todo mito é falso. Logo, toda religião é falsa?

Um mito não necessariamente é falso. Mito é uma crença que se apoderou de um povo e que ele assume como sendo a explicação para uma série de fatos. Eles não possuem embasamento em evidências e comprovações fáticas mas advém de narrativas ancestrais que são passadas de geração a geração desde tempos pré-históricos. Por coincidência, pode ser que alguma delas corresponda à verdade dos fatos. No caso das religiões contudo, nenhuma delas derivou de um mito que corresponda à realidade dos fatos. Portanto, todas as religiões são falsas.

https://www.youtube.com/watch?v=8RaGcWX5mrU Bach - Brandenburg Concerto No. 6 in B-flat major BWV 1051 - 3. Allegro. Para mim os concertos de Brandenburg são uma obra prima!

Concordo. Planejo fazer um programa sobre eles em abril. Vou ver se apresento as interpretações regidas pelo Karl Richter:
Eu tinha essa interpretação em LPs do selo Archiv, da Deutsche Grammophon, mas não sei com quem deixei. A vantagem é que os álbuns tinham todos os comentários. Era só traduzir do alemão.

"Todavia, não havendo escapatória, para se evitar o mal de muita gente, pode-se admitir o mal de alguns." No caso de tortura para que um terrorista revelasse um plano genocida e entregasse seus comparsas, seria plausível o mal deste sujeito para se evitar o mal de muitos?

Não. O que eu disse é uma situação em que, por exemplo, se tenha que impedir a ação de malfeitores para que pessoas sofram algo de ruim e nessa ação, os malfeitores venham a sofrer. Trata-se de uma legítima defesa. A questão da tortura em não admito. Mas admito o uso de algum tipo de soro da verdade ou detectores de mentira em interrogatórios.

A filosofia tem duas faces, meu caro Ernesto, uma voltada mais para a religião e arte, e outra, voltada mais para a ciência. Contudo, você acredita que um dia haverá predominância somente da filosofia voltada somente para a ciência? O empirismo está a ganhar mais adeptos ante o misticismo religioso?

Não existem essas duas faces da filosofia. A filosofia se ocupa de todo o tipo de saber humano. Religião não é um saber, a não ser quando estudado sob um prisma antropológico, sociológico ou filosófico. Em si mesmo religião é uma crendice tão despropositada quanto astrologia, numerologia, quiromancia, e esse tipo de baboseira todo. Só que adquiriu um estatuto de coisa séria e veneranda porque foi abonado pelos poderosos. Mas, no fundo, não há diferença entre o espiritismo, considerado uma crença culta, das crenças africanas e indígenas. A Filosofia pode estudar a religião e, inclusive, até, se existe ou não Deus. Mas nunca sob o prisma da fé. Nisso ela não se distingue do estudo que faz da validade da ciência ou da qualidade da arte. Filosofia é uma só. Quanto ao aspecto científico, é preciso entender que a ciência moderna não é apenas empírica. Pelo contrário, a Física Quântica e a Relatividade, por exemplo, não são quase nada empíricas. A proposta de Teoria das Cordas, então, não é nada empírica. Nem um pouquinho. Quanto ao misticismo, trata-se de algo que não tem nenhum direito de ser incorporado à Filosofia e nunca teve, como bem o mostraram William de Ockham e Duns Scoto. A Filosofia, a Sociologia e a Antropologia podem estudá-lo como um fenômeno exibido por alguns seres humanos. Mas o misticismo, em si mesmo, é uma completa fraude. Simplesmente não existe.

Em entrevista à rede televisa ABC News, o presidente dos Estados Unidos afirmou que "técnicas de tortura funcionam" e que, se preciso, ele as apoia. O que pensa sobre isso? Seria Trump um psicopata, narcisista, ou apenas um patriota que deseja evitar que aconteça um mal para muitos?

É um doido varrido, espanado, lavado, enxaguado, enxugado e lustrado. Pode até que seja um patriota. Mas um patriota fanático, como os membros do Estado Islâmico, do Al Qaeda, do Hamas, do Hezbollah, do Boko Haram, Ku-Klux-Kan e similares, que ele pretende combater. Ou um general da SS. Patriotismo fanático é o mesmo que esse tipo de terrorismo. Uma completa abominação.

Acontece que o raio do buraco negro é 1 cm e o da casca esférica é aproximadamente 15 cm. Nessa condição a curvatura espaço tempo no interior desta é hiper intenso, fazendo-a desabar sobre si mesma. Uma casca esférica não deveria anular o campo dentro de si?

A casca esférica anula o campo de sua própria massa em seu interior. Mas não o campo de algo que esteja em seu interior. E, diferentemente de uma casca metálica em relação a um campo elétrico, não anula o campo de corpos carregados externos a ela em seu interior. Um buraco negro de um centímetro de raio teria uma massa de 6,7E24 kg que é praticamente a massa da Terra. A uma distância de quinze centímetros, tal massa geraria um campo gravitacional de 2E16 N/kg e um gradiente de campo gravitacional de 2,7E17N/m.kg. Esses valores são bem difíceis de serem compensados por forças intermoleculares ou interatômicas de qualquer sistema mecânico, portanto o campo do buraco negro levaria fatalmente essa casca a cair para ele. A não ser que ela estivesse girando para que a força centrífuga anulasse a gravidade. Mas, nesse caso a velocidade angular teria que ser de trezentas e quarenta mil rotações por segundo. Note que o campo gravitacional da massa dentro da casca não é anulado pela presença da casca. Nem o campo elétrico de uma carga dentro de uma casca condutora é anulado pela casca. A casca condutora só anula o campo elétrico em seu interior devido à carga dela mesma ou a cargas externas a ela.

que autoridades pessoas como marcelo carsten, silvio zanin e tairson senna tem de falar sobre fisica por ai sendo que nem diploma ainda tem no assunto? falam até de fisica moderna (eu já li em suas respostas e blogs), mas ainda não concluiram nem a graduação na universidade, o que acha estou errado?

Não é preciso ter autoridade para falar sobre o que se quiser falar. Se eles gostam do assunto e o estudam, podem perfeitamente falar sobre ele. Isso não significa que o que eles estejam dizendo seja correto. Mas as pessoas têm o direito de falar mesmo que não seja correto. Se alguém verificar que não seja, então se manifeste, fazendo a correção. Pelo que percebo, a maior parte do que eles falam não são afirmações e sim indagações. Por outro lado, mesmo pessoas que possuam diploma, podem dizer coisas incorretas sobre o assunto em que são diplomadas. Admiro esses e essas estudantes, inclusive do Nível Médio, que se interessam por Física Moderna (Quântica, Relatividade, Astrofísica, Cosmologia), que nem caem no ENEM e nos vestibulares e, mesmo assim, eles e elas estão interessados em saber.

Me perdoe a insistência. Acho que você não compreendeu. Veja a primeira pergunta. O pequeno buraco negro está dentro da casca esférica.

Sim. Entendi perfeitamente. Um buraco negro só engole o que passa para dentro de seu horizonte de eventos. O que estiver fora, a não ser que esteja em uma órbita que leve para o interior do horizonte de eventos, continua orbitando o buraco negro como se ele fosse uma estrela qualquer. E isso pode ser tanto um planeta, quanto outra estrela, quanto uma casca em torno dele, desde que essa casca esteja em órbita e não parada.. Se estiver parada, a atração que ela sofrerá radialmente em direção ao buraco negro dependerá de sua distância a ele. Se ela for sólida (ou um tipo de treliça), tem que ver se a força gravitacional, naquela distância, poderá ser suportada pela força das partes da treliça ou do sólido entre si, que sustentam a sua estrutura. Por exemplo, se a Terra fosse um buraco negro e se construísse uma casca posicionada no lugar onde, atualmente, se situa a superfície da Terra, Uma estrutura de aço teria capacidade de suportar a atração que o buraco negro faria sobre ela que, seria a mesma que a Terra, como está, faz sobre algo que se construa sobre sua superfície.

A sabedoria deve ser usada para o bem de todos, mas como saber se o que tenho por bem não pode ser um mal para terceiros?

O critério para discernir o bem do mal não é subjetivo. Bem é uma ação que promova a satisfação, a alegria, o prazer, o lucro, a felicidade, o bem estar e tudo de bom para o maior número de seres (não só humanos). Se ela fizer isso para alguns e o contrário para muitos, não é um bem. Mal é a ação que, ao contrário, promova a insatisfação, a tristeza, a dor, o prejuízo, a infelicidade, o mal estar e tudo de ruim para alguém. Note que não estou dizendo para muita gente, basta ser par um. Todavia, não havendo escapatória, para se evitar o mal de muita gente, pode-se admitir o mal de alguns. Mas não promover o mal de alguns para o bem de muitos (note que não é a mesma coisa evitar o mal e promover o bem, pode-se ficar numa situação neutra). Não se pode fazer o mal para promover o bem, nem se pode promover o bem de alguém às custas do mal de muitos.

Filosofia é como uma árvore, cujas raízes são a Metafísica, o tronco a Física, e os ramos que saem do tronco são todas as outras ciências. O que podemos aprender com essa metáfora? Vc está de acordo com ela, ou seja, de que a filosofia é como árvore?

Essa não é uma boa analogia. O conhecimento é que poderia ser considerado uma árvore em que as raízes são a Filosofia, o tronco a Física e os ramos as demais ciências, sendo a Matemática a seiva.

Já vi gente dizendo que a prostituição é algo essencial para sociedade, pois há pessoas que nunca conseguiriam sexo, se não fosse por meio pago. O que acha disso?

Acho que tem muitas pessoas que não conseguem sexo por um acordo consensual com outra pessoa e, simplesmente, ficam sem sexo. Isso é muito comum com pessoas idosas e mesmo não idosas que ficam viúvos ou, principalmente, viúvas. Muitas delas e até deles não acham apropriado se envolver com prostitutos ou com prostitutas. No caso de moças e moços muito feios mas carentes de sexo, que não conseguem atrair ninguém, acho que seria válido existir uma espécie de fraternidade (como se fosse uma ordem religiosa, mas não religiosa) de pessoas de ambos os sexos, que, por caridade, fariam sexo com essas pessoas, para suprir a carência delas.

Discordo! Se a bola menor for um buraco negro, não há casca esférica que o resista. Estou errado?

Está. Se a casca estiver fora do horizonte de eventos do buraco negro ela pode, perfeitamente, ficar em órbita estável em torno dele sem cair para ele.

É melhor estudar múltiplas matérias em um mesmo dia ou estudá-las em dias alternados?

Depende. Se você é estudante do Curso Médio, é importante que você estude, todo dia, as matérias que foram dadas nas aulas daquele dia, antes que durma, em um outro momento do dia. Mas se você está estudando por conta própria e não assiste aula de nada, Pode se focar em duas matérias por dia. Uma só é ruim, porque esgota o cérebro de um mesmo assunto e ele começa a ficar refratário ao conteúdo. A não ser que seu tempo de estudo seja pouco. O ideal seria estudar quatro horas por dia de cada uma, em dois turnos de duas horas de cada uma, intercalados e separados pelo almoço ou pelo jantar. E, inclusive, ao fim de cada hora, é bom fazer um descanso de uns onze minutos. Mas se você tiver só quatro horas por dia para estudar, é melhor estudar uma só, porém em dois períodos separados por um intervalo de umas duas horas, pelo menos. Sempre divida o tempo de estudo em um período teórico e outro prático, não emendados. No teórico é bom que se faça um resumo do que se está estudando, como se estivesse escrevendo uma apostila para outros estudarem por ela. Nisso você vai ter que consultar os textos para desenvolver o seu próprio.

https://ask.fm/wolfedler/answers/140588043549. Então, é melhor deixar de lado esses livros que falei e me focar nos que você me indicou?

Depende. Quanto tempo falta para você fazer o ENEM? Se ainda faltam três anos, é bom estudar pelo Bezerra e pelo Carvalho. Mas se falta só um ano, é melhor estudar por esses livros atuais mais voltados para o ENEM, mesmo que você aprenda menos matemática para o que vai ser importante na vida profissional. Mas pode ajudar a passar no ENEM.

Amizade verdadeira acaba ?

Pode acabar sim. Certamente acaba com a morte. Mas pode durar até a morte ou acabar antes. Não há nada no fato de ser verdadeira que implique que seja permanente. Desentendimentos graves podem ocorrer que a terminem. Por exemplo, duas amigas se apaixonarem pela mesma pessoa e essa pessoa escolher uma delas ao invés das duas em paralelo, para estabelecer uma relação amorosa (namoro e, até, casamento). Fora outros assuntos, dentre os quais um dos que mais desfazem amizades são os financeiros.

Uma bola de futebol, funciona como uma casca esférica. Se colocarmos no interior dela uma bola de gude de 1 cm de raio, podemos então afirmar que uma bola não consegue atrair gravitacionalmente a outra, certo? Mesmo que a bola menor seja (muito hipoteticamente) um buraco negro?

Sim. isso mesmo. Todavia isso só seria percebido se ambas estivessem bem afastadas de qualquer outro corpo, como o planeta Terra. Nesse caso a atração da gravidade da Terra (que penetra no interior da bola grande) é tão maior do que a atração mútua entre as bolas (que elas exibiriam se estivessem do lado de fora uma da outra), que dá para se perceber. Talvez em um experimento bem sensível em que ambas as bolas tivessem uma massa razoável de muitos quilogramas, como na experiência de Cavendish.

Qual a diferença de habilidade entre um QI de 135 e um de 140? Se é que tem tanta diferença.

A diferença existe e é demonstrada pela quantidade de respostas certas que uma pessoa de QI 140 dá a mais em um teste de QI do que uma pessoa de QI 135. Isso significa que, aproximadamente, uma pessoa de QI 140 tem 140/135 vezes mais habilidade em solucionar corretamente e mais rapidamente problemas, do que uma pessoa de QI 135.

https://youtu.be/f_dcm82XHqM Que resposta simples daria pra refutar esse que se autodenomina "filósofo" falando sobre deus e causa do universo?

De fato o Universo contém força, energia e magnetismo. Mas não é nada do que ele, e outros, chamam de "força", "energia" e "magnetismo". Esse pessoal chama de "força" algum agente volitivo que promoveria as ações que levariam aos efeitos observados. Ora, força, fisicamente, é meramente a grandeza definida para medir a intensidade das interações entre os sistemas e não tem nada volitivo. Essas interações ocorrem espontaneamente em razão na natureza e das propriedades dos sistemas, bem como em função de seus estados de movimento e de suas posições relativas. Estas, por sua vez, advém de movimentos e posições anteriores e assim por diante, não havendo nada que planeje e determine que seja de tal ou qual modo. "Energia" para eles é uma espécie de "disposição" dos sistemas (que são subconjuntos do Universo) em promover alterações e realizar ações. Energia, fisicamente, não é nada disso. Trata-se de uma propriedade dos sistemas e de suas interações, advinda de seus movimentos e posicionamentos, que exibe a notável propriedade de se manter constante nas transformações ocorridas com os sistemas (se se incluir todas as modalidades) e que, portanto, permite estabelecer o leque de possibilidades futuras para os estados para os quais os sistemas evoluirão ou, pelo menos, estabelecer as situações impossíveis de virem a se dar. E "Magnetismo" é algo por eles consideram como uma manifestação de uma espécie de "atração", que os sistemas apresentam pelo fato de estarem imbuídos de boas intenções. Todavia, magnetismo, fisicamente, é apenas um fenômeno exibido por cargas elétricas em movimento relativo que se sobrepõe aos fenômenos elétricos que os sistemas possuidores de carga elétrica exibem quer estejam ou não em movimento. Claro que ateus e crentes admitem a existência de força, energia e magnetismo no sentido físico que descrevi e, inclusive, não se trata de nada em que se "acreditar", pois é uma constatação, decorrente da definição dos termos. Mas nenhum ateu e, mesmo, muitos crentes, não admitem a existência de "força", "energia" e "magnetismo" no sentido esotérico que ele apresenta. Isso não existe mesmo e, mesmo que existisse, não teria nada a ver com o conceito de "Deus", que é o conceito de uma entidade, que poderia, no caso, ser o agente dessa "força", o fornecedor dessa "energia" ou o estabelecedor desse "magnetismo". Mas nada disso existe e, portanto, não são justificativas para considerar a existência de algum "Deus". Inclusive poderiam, até, existir, sem que existisse Deus nenhum. O que não significa que Deus não exista, mas não por essas razões. De fato, não existe, simplesmente porque não há razão nenhuma que implique que deva existir.

É possível fazer o qi passar de 100 para 135?Ou é quase impossível?

É quase impossível. Eu diria que, no máximo, se consegue chegar a uns 115 ou 120. O aprimoramento do QI por meio de exercícios neuróbicos e treinamento em geral se faz sobre uma base genética pré-existente, que não se consegue modificar. Mas vale tentar, mesmo que seja bem custoso.

Estou estudando pelos livros de matemática do Thales M. Carvalho e Manoel J. Bezerra, porem, também vou fazer o ENEM. Continuar estudando por esse livros e depois complementar com alguns outros assuntos que cai no ENEM e, que não possuem nesse livros, seria uma boa escolha?

A questão é a seguinte: Sai-se bem no ENEM a pessoa que já sabe a matéria por tê-la aprendido ao longo de todos os anos do ensino fundamental e do médio. Isto é, quem foi sempre um bom aluno. Os livros do Carvalho e do Bezerra são dois dos que melhor preparam o aluno para, de fato, saber matemática. Mas eles foram projetados para serem estudados ao longo de três anos. Quem não aprendeu o que era para ser aprendido nesses três anos, vai ter dificuldade para assimilar tudo em um ano só. Nesse caso, como uma muleta, as apostilas dos cursinhos podem ajudar. Mas elas não garantem sucesso e nem conseguem suprir o que ficou em falta ao longo do tempo de alguém que não foi sempre um bom aluno. Há livros atuais de matemática para o Nível Médio em um só volume que também se prestam a isso razoavelmente. Recomendo o do Iezzi e Dolce.

Gostas de ver desenhos animados? Qual é o teu favorito?

Sim: Tom & Jerry, Popeye, O Pica Pau, Pernalonga, MIckey, Pateta, Pluto, Pato Donald.

Professor, algum conselho pra não se esquecer dos "estudos"? Porque assisti o seu canal mês passado, aprendi muita coisa, porem eu me esqueci de 80% do que falou sendo só um mês que tenha passado.

Isso porque você só usou os sentidos da visão e da audição e não se dedicou a assimilar, de fato, o conteúdo do vídeo. Para não esquecer você teria que, ainda no mesmo dia, em outro momento, redigir um trabalho sobre o vídeo, por escrito, revendo cada parte para colocar no texto com suas próprias palavras. E depois ensaiar uma apresentação como se você estivesse dando uma aula sobre o conteúdo do vídeo. É assim que se aprende. Preparando-se para dar uma aula sobre o assunto. Mas tem que fazer isso tudo no mesmo dia, antes que durma, mas em um momento separado do primeiro, para que o seu cérebro confira relevância ao conteúdo registrado na memória de curta duração e passe para a memória de longa duração. Não pense que aprender algo, pra valer mesmo, seja fácil. Não é. Requer muito empenho.

Ernesto, como faço para perdoar? Como não sentir mais raiva de uma pessoa? Às vezes sou muito rancoroso.

Toda mudança de concepção e de modo de pensar começa, justamente, pensando-se muito sobre o assunto, refletindo e vendo o que é o certo a fazer. Faça a meditação monástica ocidental que, ao invés da budista, não leva a mente a afastar qualquer pensamento mas, ao contrario, leva a mente a ficar pensando sobre um um mesmo assunto. Medite sobre sua condição rancorosa todo dia, por um terço de hora e vai construindo em sua mente a concepção oposta, raciocinando como ser rancoroso é algo ruim. E sempre que se sentir rancoroso, lembre do que meditou a respeito e procure agir para afastar o rancor. Isso não muda em pouco tempo mas, com insistência, muda.

Em um de seus artigos você afirma que "parte da escalada da criminalidade hoje observada no Brasil e no mundo se deve a perda desse temor do inferno". Acredito que esta explicação seja simplista pois existem muitos outros fatores como péssima distribuição de renda, falta de escolas/creches, etc

Por isso é que eu disse "parte".

(cont.)Mesmo você sempre se refere 'a presença dos seus pais em casa lhe ensinando desde a mais tenra idade e até lhe fornecendo livros e conversando com você. A maioria desta geração não possui pais com tempo de ficarem em casa, já que trabalham muitas horas e chegam extenuados em casa.

Meus pais, como funcionários públicos, trabalhavam do meio dia às seis da tarde. As noites é que eram dedicadas a estudar, a conversar, a ler, já que não havia televisão. No máximo se ouvia novelas e jornais no rádio. À noite se tocava piano, se liam histórias, se fazia o dever de casa. Isso porque tinhamos aula à tarde e podíamos acordar tarde e, portanto, dormir tarde. De manhã era a hora de brincar e de ir andar de velocípede ou bicicleta, ou brincar de pique no "Jardim do Globo", que era perto de onde morávamos. Eu gostava muito de brincar em casa mesmo, montando jogos de mecânica ou brincando de carrinhos, com minhas irmãs. Ou brincando de boneca com elas, já que meus pais eram bem avançados e liberais e achavam que era bom meninos também brincarem de bonecas e meninas brincarem de carrinho.

Se você não tivesse ancestrais da nobreza europeia, vc acha que teria seu nível educacional e cultural? Os antepassados de uma pessoa tem forte influência em seu nível educacional?

De fato o ambiente cultural e intelectualizado de minha família, tanto paterna quanto materna, há muitas gerações, faz uma grande diferença nos valores que são considerados mais elevados na educação dos filhos. Ser criado em um ambiente em que, não só em casa, mas na casa de todos os tios e avós, há vastas bibliotecas, onde todo mundo fala o português correto, onde os modos de ser e de ser portar são modos refinados, onde se sabe como se portar com elegância, onde se sabe usar os talheres corretos, onde as moças todas tocam piano, que se tem em casa, e de cauda. Tudo isso, mesmo que no seio de famílias que já não possuem poder nenhum e tampouco sejam ricas, faz uma grande diferença nas escolhas, nos assuntos das conversas, no gosto artístico, musical, literário, nos ideais, nas escolhas profissionais, nas concepções ideológicas e tudo o mais. O interessante é que, minhas famílias, apesar de tradicionais, são famílias rebeldes, no sentido, por exemplo, de todos eram abolicionistas e republicanos, no tempo do Império, tanto no Brasil quanto na Áustria (se bem que na Áustria não havia escravos). Eram de tendências socialistas e comunistas, isto é, de esquerda, o quanto se poderia se considerar ser de esquerda antigamente. Votavam em partidos de esquerda ou centro esquerda. Meu pai era do antigo PSD e opositor da antiga UDN. Minha família era toda urbana, sem nenhum vínculo rural, exceto duas primas do meu pai que se casaram com fazendeiros. Em geral eram advogados, professores, médicos, engenheiros, funcionários públicos, militares. Um ou outro era comerciante. E as mulheres, quase todas, trabalhavam fora, numa época em que isso era incomum. Isso tudo, realmente, faz diferença na educação que eu tive, por exemplo.

Professor, qual a sua opnião sobre a educação financeira. Na sua opnião, você acha que é importante manter poupança?

Como um anarco-comunista sou totalmente contra a existência do dinheiro. Acho que tudo deva ser de graça e que se deva trabalhar inteiramente de graça. Consoante minhas concepções, jamais juntei dinheiro nenhum, não tenho bem nenhum, todo o dinheiro que tenho eu gasto. Não acho importante nada que seja relativo a finanças. Só não dispenso totalmente meu salário porque os outros não me dão tudo o que eu preciso de graça. Mas tenho só um carro velho que não vale nem uns sete mil reais. Não tenho casa própria, vivo na de minha mulher. Grande parte de todo o trabalho que faço eu faço de graça. Essa é a minha concepção e assim é que eu acho que o mundo deva ser.

Preferias explorar o espaço ou os oceanos?

O espaço. Questão de gosto.

Concordo com a sua resposta sobre Mecânica dos Fluidos. O que eu quis dizer é que, na prática e no Brasil, os cursos de engenharia pelo menos parecem discutir bem mais a própria Física de Fluidos, enquanto os cursos de Física tratam o assunto como uma lateralidade.

Sim, é fato. Mesmo as engenharias, somente a hidráulica, a térmica, a naval, a mecânica, a aeronáutica e a aeroespacial é que cuidam mais profundamente do tema. No caso da Física, o assunto é visto por quem for se dedicar a astrofísica, cosmologia, física intragaláctica, meteorologia, oceanografia e alguns casos especiais.

Professor, alega-se que o cristianismo é importante pelos fundamentos morais (amor ao próximo, força para encarar os problemas cotidianos). Como estabelecer um referencial moral mais elevado, que pregue a empatia e a ética sem precisar de figuras divinas? Como seria uma moral científica, humanista?

Os preceitos morais não são uma exclusividade do cristianismo. Todas as religiões os possuem. Em verdade elas os apropriaram da humanidade. Leia os artigos que escrevi sobre moral, ética, religião e ateísmo:
http://wolfedler.blogspot.com.br/search?q=%C3%A9tica+ate%C3%ADsta
http://www.ruckert.pro.br/blog/index.php?s=%C3%A9tica+ate%C3%ADsta
http://wolfedler.blogspot.com.br/search?q=Moral+e+%C3%A9tica
http://www.ruckert.pro.br/blog/index.php?s=moral+e+%C3%A9tica

Por que mesclar Deus com os objetos da pesquisa científica ortodoxa horroriza hoje as mentes acadêmicas? Não poderia existir faculdade de criacionismo somente para aqueles que estudam essa teoria? O que o professor pensa disso, e por que essa mistura poderia ser danosa?

Porque o criacionismo não tem base científica nenhuma. É uma modo forçado de fazer com que a crença de que os seres todos, inanimados e vivos, ao invés de terem surgido por evolução, foram criados, cada um, diretamente por Deus. Isso é MENTIRA. A evolução é sobejamente comprovada e o criacionismo não tem comprovação nenhuma. Não tem escapatória. A noção de qualquer interveniência divina é incompatível com o estudo sério dos fatos científicos.

Professor, tenho impressão de que no curso de Engenharia Mecância se aprende mais sobre Mecânica dos Fluidos do que na Física. O que acha?

Depende do aspecto. Na Engenharia se estuda a mecânica dos fluídos para se usar em máquinas. Na Física se estuda a mecânica dos fluídos para se aplicar nos movimentos atmosféricos, oceânicos, galácticos, no interior dos planetas, no interior das estrelas, nas nebulosas e coisas assim. De modo que a mecânica dos fluídos da física é bem mais abrangente e complexa, pois a da engenharia é um caso particular e especial apenas.

Alguém que chega ao comunismo por achar que essa é a justiça divina e que Jesus era de certo modo, ainda é melhor que um ateu de direita?

O fato de ser ateu ou crente e de ser de direita ou esquerda são totalmente independentes. Sou um ateu de esquerda. Discordo dos crentes e discordo dos direitistas. Como discordo dos esquerdistas estatistas e dos niilistas. Mas acho que Jesus deu muitos ensinamentos proveitosos, se deixarmos de lado a questão dele se considerar deus e de que sua morte redimiu a humanidade, bem como de que nasceu de uma virgem e que ressuscitou. Isso é balela. Mas a parte social de seus preceitos são muito válidas e bem comunistas mesmo. Acho que todo mundo tem que buscar a santidade e nisso seguir muitos preceitos cristãos, bem como budistas, hinduístas e, até, alguns muçulmanos. Desde que deixe tudo o que seja sobrenatural de fora. Mas a determinação em ser bom, fazer o bem, ser justo, honesto, generoso, sincero, bondoso, solidário, compassivo, tolerante, combater o mal, isso está na maioria das religiões e é ótimo, inclusive não sendo preciso ter religião nenhuma para seguir. O problema em acreditar em Deus e na existência de uma alma imortal é simplesmente que é uma mentira. Da mesma forma que achar que exista céu, inferno, anjos e demônios.

Se a moral não é um fenômeno que pode ser pesquisado por meio da ciência, como podemos dizer que há moral objetiva?

Mas não há moral absoluta. Pode-se dizer que ela seja objetiva no sentido de que suas prescrições, permissões e proibições não se aplicam individualmente ao sujeito em particular (o que seria subjetivo), mas objetivamente a todo um grupo a que pertençam os sujeitos que são o objeto dessa moral. O que é absoluto é a ética, esta sim, que pode ser estudada filosoficamente de modo objetivo. Não cientificamente porque o assunto não se adequa ao método científico e sim ao filosófico. A moral, por sua vez, pode ser estudada sociologicamente como um fenômeno, mas não o conteúdo de suas prescrições, permissões e proibições. Esses conteúdos são convencionais e arbitrados pelo grupo dominante de algum estrato social em dado lugar e dada época. Ou seja o conteúdo do que a moral prescreve, permite ou proíbe não é passível de ser deduzido de modo lógico e racional. O que não acontece com a ética. O ideal é que a moral seja sempre norteada pela ética. Todavia, em muitos casos, isso não se dá, ficando a moral como a expressão da vontade da classe dominante do grupamento humano considerado. Vontade essa que, muitas vezes, é a de uma única pessoa (um ditador, como Hitler).

Ernesto, qual cálculo os astrônomos usam para saber quando irá ocorrer um eclipse? Aproveitando, você viu o eclipse que ocorreu hoje? Foi muito bonito!

Esqueci completamente de ver. Quanto aos cálculos, são bem trabalhosos. A explicação seria um livro.
http://astronomy.stackexchange.com/questions/231/what-is-the-formula-to-predict-lunar-and-solar-eclipses-accurately
https://eclipse.gsfc.nasa.gov/SEmono/reference/explain.html
https://eclipse.gsfc.nasa.gov/lunar.html
https://archive.org/stream/mathematicaltheo00buchuoft/mathematicaltheo00buchuoft_djvu.txt
http://www.mreclipse.com/Special/SEprimer.html
https://en.wikipedia.org/wiki/Solar_eclipse
https://en.wikipedia.org/wiki/Eclipse_cycle
https://en.wikipedia.org/wiki/Lunar_eclipse
http://www.mreclipse.com/Special/LEprimer.html

considerando que um eletron ou um quark podem ser ondas, eles podem se mover mesmo assim? esclareça-me por favor. e a funçao de onda das particulas e valida apenas para a materia?

A função de onda é uma descrição matemática da intensidade do campo da matéria e de todo constituinte físico do Universo. Quarks, léptons (como os elétrons e os neutrinos), glúons, fótons, todo constituinte físico do Universo, bem como seus conglomerados, como prótons, nêutrons, núcleos e até o Universo inteiro tem uma função de onda. O movimento de uma partícula é o movimento do máximo da envoltória dessa função, que envolve o grupo de ondas superpostas que representa a distribuição da intensidade do campo cuja quantização aquela partícula é. A velocidade com que a partícula se move é, justamente, a velocidade com que esse grupo se move, chamada "velocidade de grupo". O campo da matéria (que por extensão da palavra também se aplica ao campo bosônico dos mediadores de interação) tem uma ondulação que se propaga com uma envoltória cuja velocidade de deslocamento é a velocidade da partícula. Ou seja, a partícula "É" o grupo de onda quantizado do campo que lhe corresponde.

Professor, sobre o "tecido" espaço-tempo, tenho uma dúvida... sempre estamos em movimento no espaço, pois a terra gira em torno do sol, o sol em torno da galáxia e a galáxia também está em movimento, minha dúvida é: se conseguíssemos realmente parar no espaço o que aconteceria com o tempo?

O tempo advém do fato de que o estado global do Universo está em mutação. Não há como se dizer que se está parado em relação ao espaço porque o espaço não é algo que possa ser usado como uma referência. O que pode acontecer é que tudo fique parado, cada coisa em relação às outras. Se tudo mesmo parar, isto é, os elétrons pararem de se mover nos átomos, a luz parar de se propagar para todos os lados que se propaga, então, realmente, se dará a interrupção da passagem do tempo. Isso pode acontecer e, nessas interrupções, nada acontece. Quando ela acabar, o fluxo normal de tudo prossegue como se nada houvesse ocorrido, Inclusive, desde que você começou a ler este texto, podem ter ocorrido milhões de interrupções do fluxo do tempo.

Pessoas analfabetas científicas dizem por aí que a física quântica PROVA que o universo é uma ilusão, ou q a teoria da evolução PROVA que o homem veio do macaco... que é exatamente uma teoria ou uma prova científica? (sei que isso é o básico de filosofia da ciência, mas muitas pessoas não sabem)

Para começar, o Universo NÃO É uma ilusão e, portanto, a Física Quântica não tem como provar que o seja. E, mesmo que o fosse, a Física Quântica não é que provaria. A Física Quântica é uma ciência que estuda e estabelece um modelo descritivo dos fenômenos naturais em nível atômico e sub-atômico, em que o modelamento propiciado pela Física Clássica, desenvolvida por Newton e vários outros, prevê resultados que não condizem com os realmente observados na natureza, o que a Física Quântica faz. Seja o Universo uma ilusão ou não, a Física Quântica, do mesmo modo que a Física Clássica, se apoia nas observações capturadas pelos sentidos humanos nus ou assessorados por instrumentos (microscópios, telescópios, termômetros e qualquer tipo de medidor). Se essas informações são provenientes de entidades e fenômenos reais ou ilusórios é algo que não se pode saber com os instrumentos científicos. A ciência toda, contudo, é construída na suposição de que o mundo objetivo, exterior às mentes, é real. Por isso é que dito que o Universo NÃO É uma ilusão. Essa é a hipótese de trabalho básica da ciência.

quando se define alguma coisa em fisica, ja se esta aderindo a filosofia? em que parte a ontologia entra na fisica?

Sim, porque a ontologia, justamente, cuida do que as coisas são, isto é, o que algo tem de característico para ter algum nome. A essência é, justamente, a propriedade ou propriedades, sem as quais aquilo não é o que é. Os acidentes não comprometem o fato daquilo ser o que for. Então, ao se dar uma definição de algo em qualquer ciência, está se dizendo o que, em função das propriedades que possui, pode receber aquele nome. Isso é, justamente, ontologia. Além disso, a Física é fenomenológica, isto é, o que quer que seja que tenha recebido algum nome se relaciona com outras coisas, relações, ou o que seja que também tem outros nomes. A busca dessas relações é que é o âmago da ciência em si mesma. Ela poderia ser edificada com base em definições inteiramente diferentes. Definições são arbitrárias. Mas as relações não. Elas descrevem o modo como a natureza se comporta, que não depende da vontade de ninguém. Para estudar a fenomenologia da ciência (que, afinal, é a própria ciência) faz-se uso de modelos descritivos (especialmente matemáticos) que propiciam explicações, bem como previsões. Se as previsões se confirmam, o modelo se torna a teoria explicativa do fenômeno. Isso é epistemologia.

Ernesto, qual a diferença entre ser intelectual e ser intelectualizado?

A diferença é que o intelectual realmente conhece com profundidade e amplitude alguns assuntos em que seja versado e é capaz de entabular longas conversas sobre eles, municiando-se de argumentos consistentes e eficazes sem precisar de se valer de consultas, exceto em alguns casos particularíssimos. O intelectualizado ou intelectualoide, tem uma ideia dos assuntos que aborda mas não os conhece em amplitude e profundidade, acabando por se atrapalhar ao entabular alguma discussão a respeito, por falta de embasamento. O intelectual, mesmo quando não esteja seguro de algo, sempre sabe onde recorrer para esclarecer, enquanto o intelectualoide não.

Eu não abrevio EDO, é pq o espaço da pergunta é limitado. Tento abreviar o máximo de palavras pra expor melhor. Sobre o artigo, não precisa ser nada inédito? Pois ele quer uma aplicação.

Uma ótima aplicação das equações diferenciais ordinárias é o estudo das oscilações livres, amortecidas e forçadas. Você acha isso em qualquer livro de mecânica clássica vetorial (não nos livros de física geral). Ou então o estudos dos circuitos elétricos que envolvam resistência, capacitância e indutância. Outra sugestão é a solução do problema gravitacional de dois corpos, que mostra que as órbitas são seções cônicas. No caso é preciso se trabalhar com coordenadas polares. Um bom livro que trata disso é o de mecânica do Symon. Outra aplicação já é em física quântica, não só na solução dos orbitais do átomo de hidrogênio, mas também nos problemas de espalhamento, como a experiência de Rutherford. O átomo de hidrogênio é meio avançado, porque lida com equações diferenciais ordinárias lineares com coeficientes não constantes, o que leva ao método de Frobenius e às funções especiais (polinômios de Laplace e polinômios de Laguerre).

Os teus pais gostam da música que ouves?

Quando eles eram vivos gostavam, pois, inclusive foram eles que me incentivaram e cultivaram em mim o apreço pela música clássica, que sempre era ouvida em nossa casa nos antigos discos de baquelite, de 78 rotações por minutos, que ainda conservo e, depois, nos long-playings. Além de me terem incentivado a estudar piano, que minha mãe tocava. Além disso, meu pai me desenvolveu o gosto pelo Jazz, pelo Blues, pelo Tango, pelo Samba, pela Easy Music, pelas canções francesas, italianas, mexicanas, espanholas, napolitanas, pela ópera, pelo bolero, pela Bossa Nova e a música popular brasileira de qualidade em geral.

Telecinesia. Para um objeto em repouso se mover, uma força precisa agir sobre ele. Se a força vem da mente, esta mente não é abstrata e sim física. Sendo física pode ser medida de alguma forma. Logo a telecinesia é uma bobagem monstruosa. Estou certo ou falei asneiras?

Está certíssimo! Telecinesia é uma falácia. Não existe. É completamente impossível. Os casos mostrados são todos fraudulentos. Quem acredita nesse tipo de bobeira, como em telepatia, projeção astral, clarividência, anjos da guarda, premonição e esse tipo de coisa é uma pessoa crédula e ingênua que não tem o menor conhecimento sobre como a natureza funciona e como qualquer manifestação sobrenatural é, meramente, uma ilusão. Não existe nenhuma realidade objetiva, exceto a realidade física. Não existe nenhum tipo de entidade sobrenatural: nem deuses, nem espíritos, nem almas, nem anjos, nem demônios, nem gênios, nem qualquer tipo de elemental.

por que na fissao nuclear como a das bombas atomicas ocorre o que se chama de "lixo radioativo", de onde surge esses elementos instaveis apos a explosao da bmba por exemplo sendo que os elementos instaveis fissionaram e ficaram estaveis?

Porque o aproveitamento da fusão nuclear não é de 100%, havendo uma razoável porção do combustível radioativo que não se fissiona totalmente mas que emite um ou dois nêutrons, transformando-se em isótopos instáveis.
Leia estes artigos:
http://www.fisica.ufpb.br/~jgallas/CURSOS/Estrutura02/energia_nuclear_cap43.pdf
https://pt.wikipedia.org/wiki/Bomba_nuclear

mas entao a definiçao de que a radiaçao seja um conglomerado de bosons se aplica?

Radiação pode ter dois significados. Em radioatividade é o fluxo que emana de uma fonte radioativa e pode ser de três tipos: alfa (que são bósons), beta (que são férmions) e gama (que são bósons). Mas o significado de radiação como um dos constituintes substanciais do universo é outro. Nesse caso radiação é a entidade que intermedeia as interações. Então elas são todas bósons, como os fótons (de que são constituídas as radiações gama, a luz visível e as ondas de rádio), os glúons (que só atuam dentro dos núcleons - entre os núcleons a interação forte é mediada por mésons, que também são bósons não elementares) e os bósons W e Z, que medeiam a interação fraca. Os grávitons são hipotéticos e os bósons de Higgs não medeiam nenhuma interação conhecida. Por enquanto é o que se sabe (pelo menos o que eu sei).

Ernesto, meu professor de EDO quer que nós façamos um artigo sobre EDO, mas um artigo precisa ser algo inédito, certo? Não faço ideia do que escrever, até pq nao tenho o conhecimento completo de EDO. A nivel de um artigo cientifico, acho que não sairá bom. Algum tema como sugestão?

Você tem que estudar vários livros sobre Equações Diferenciais Ordinárias (não use abreviações: é deselegante) e, então, na sua cabeça, montar uma forma de apresentar o assunto e explicar a quem for ler para que aprenda com o seu artigo, como se fosse um capítulo de um livro. Isso dispende várias dezenas de horas de trabalho.

o que é mais importante a um fisico, lembrar de toda equaçao para explicar tudo a sua volta ou apenas saber explicar conceitualmente e se n lembrar da equaçao, consultar em algum livro ou artigo?

Entender conceitualmente e fenomenologicamente é muito mais importante do que saber as equações de cor. Mas tem que saber como resolvê-las.

na sua distinçao de materia e radiaçao, vc diz q um é um sistema de fermions e outro de bosons. mas os mésons sao bosons e sistemas feitos de fermions. entao sao materia ou radiaçao? qual e a diferença então, de matéria e radiação?

Os mésons não são partículas elementares. Nesse caso eles podem ser matéria e serem bósons, como as partículas alfa. As partículas elementares que são bósons não são matéria e sim mensageiras de interação. As partículas elementares que são férmions são constituintes essenciais da matéria (que também contém bósons elementares). Os bósons elementares são os fótons, os glúons, os W, os Z, o de Higgs e o pretenso gráviton. Os férmions elementares são os quarks, os elétrons, müons e táons e seus neutrinos. E todas as suas antipartículas. Os bósons elementares, exceto o W, não possuem antipartículas.

Este sistema ajudava na conservação dos mesmos. Não sei se continua assim,lá. Mas isto foi há quarenta e seis anos atrás. Acredito que estejamos uns duzentos anos atrás dos Estados Unidos em matéria de Educação. Como atingir a estes patamares aqui no Brasil?

O Brasil está atrasado em relação aos Estados Unidos sim, mas nem tanto, porque os Estados Unidos também estão atrasados em educação. Os países mais avançados são a Finlândia, a Coréia do Sul, Austrália, Canadá, Noruega. Os Estados Unidos estão lá para o vigésimo lugar e o Brasil, quase no septuagésimo.

sexta-feira, 11 de maio de 2018

O método de Newton para resolução de equações não lineares é prático? Acho que consegui deduzir derivação/integração a partir do que vi o cara fazer no método, estou viajando, ou dá pra realmente deduzir isto? rs

O método de Newton é numérico e, em muitos casos, se requer uma solução analítica. Há outros métodos, como o espalhamento inverso e sempre se pode recorrer ao método de perturbações.

Como estudante de intercâmbio, nos Estados Unidos, constatei que os livros,todos encadernados, eram fornecidos pelo colégio e devolvidos ao final do semestre. Só levava para casa aqueles que precisava, a cada dia, no ônibus escolar. Fora disto, permaneciam no armário fornecido pela escola.

Isso é muito bom. Mas eu gosto de ter todos os livros de que preciso sempre a meu dispor a qualquer momento. Porque não tenho hora certa para estudar. Além do mais nunca me atenho a um livro só para estudar qualquer coisa. Acho importante que todo estudante construa a sua biblioteca privada para a vida toda. Não consigo entender pessoas que, ao acabar um curso, dispõem de seus livros. Se eu tivesse esses armários da escola eu iria ter dois de cada livro que ficam no armário ou então eu os levava e trazia todos, todo dia, da casa para a escola e vice versa.

Ernesto, acredita que o trabalho intelectual, a responsabilidade e os riscos possam ser assumidos por todas as pessoas? Não acha que, em uma economia planificada, potenciais empreendedores com disposições e requisitos — estes escassos — iriam empregar esforço e gerar valor? Somos seres competitivos.

Isso é que a educação tem que transformar. Nos mudar de competitivos para colaboradores. Isso faz parte do esforço civilizatório da humanidade. Muito já foi conseguido, no sentido de deixar a belicosidade e abraçar a pacificação. De abolir a intolerância, o preconceito. De aceitar as diferenças de vários aspectos culturais. De encarar os gêneros como iguais. Falta muito, é claro, como encarar livremente a possibilidade do poliamorismo e várias outras coisas. Então, essa questão das competências tem que ser encarada como qualificações que as pessoas possuidoras colocam à disposição de todos para o bem geral. Doando o seu talento para proveito de todos. E todos doam, cada um o que for capaz, para o bem dos outros. Uns são mais capazes, outros menos capazes. Nem por isso os mais capazes merecem mais do que os menos capazes. Todos doam o máximo de si e recebem o máximo dos outros, para o bem geral. Isto é que é a harmonia social da anarquia. Calcada principalmente no altruísmo. Que será atingido pelo processo educativo.

Sobre a incompatibilidade entre a ciência e a filosofia.

Agora estou me lembrando do que dissera antanho. Filosofia e Ciência são formas distintas de abordar a realidade e, em geral, se dedicam a aspectos disjuntos dela. Todavia há casos, historicamente, em que ambas se debruçaram sobre um mesmo tema. Como ocorreu com o sistema planetário. A questão se deveu a um equivoco de interpretação de que o tema fosse filosófico, quando é científico. Então filósofos fizeram proposições e, sobre elas, construíram, cientificamente, consequências que, até, se mostraram de acordo com os fatos, como o Sistema Ptolomaico. O próprio Copérnico propôs o seu filosoficamente e tirou consequências científicas que também se mostraram de acordo com os fatos, se se procedessem correções análogas às do sistema de Ptolomeu. Somente depois que Newton colocou o tema sob uma égide completamente científica e Kepler, apontou as correções observacionais que a teoria de Newton permitia encampar, é que o tema deixou de ser, definitivamente, filosófico. Nesses casos, quando houver discrepância entre a concepção filosófica e a científica, a científica tem que ser adotada, pois a ciência tem critérios seguros de verificação da validade de suas proposições, enquanto a filosofia baseia-se inteiramente no raciocínio, que pode ser falho. Isso não significa, em absoluto, que se deva abandonar a filosofia, inclusive porque, vários temas, não são passíveis de verificação empírica, ficando fora do escopo da ciência. Então é só a Filosofia que pode considerá-los. Mas ela também tem que submeter suas proposições a um crivo de testes lógicos para validá-los. A própria definição dos critérios de validação da ciência, é filosófica.

Por que a maioria dos livros de economia são lixo?

Não sei, porque, em verdade, não leio muitos livros de economia, já que meu interesse em economia é filosófico e a maioria dos livros que encontro é prático, o que não me interessa, em absoluto.

Uma nuvem só se forma se chover ?

Não. Aliás a maior parte das nuvens se forma e se dissolve sem acarretar chuva nenhuma.

Então o senhor não é marxista?

De modo nenhum. Sou um comunista anti-marxista. Marx deu grandes contribuições ao comunismo mas equivocou-se totalmente em considerar que o caminho para o comunismo fosse o socialismo de estado de "Ditadura do Proletariado" de partido único. Isso é pior do que o capitalismo. O comunismo tem que ser alcançado por uma evolução por meio de reformas que distribuam o capital entre os trabalhadores até que não haja nenhum empregado assalariado e todos sejam sócios, detentores de parte do capital das empresas. A pressão nesse sentido pode e deve ser feita por meio de greves gerais de categorias abrangendo todas as empresas do país ao mesmo tempo. E que não sejam encerradas até que se alcance o objetivo da participação societária dos trabalhadores. Que se abula o proletariado, totalmente convertido em burguesia. E que se abula, também, a aristocracia e a plutocracia. Enfim, que todos sejam burgueses.

quinta-feira, 10 de maio de 2018

Por que nas festas do carnaval europeu não há muitas brigas e sexo entre as pessoas como nas do Brasil?

Porque o povo brasileiro, de modo geral, não tem civilização. É mal-educado. É mal-comportado. É bagunceiro. É licencioso. É desrespeitoso. É depravado. E isso é culpa das escolas que se recusam a assumir o papel dos pais na educação dos filhos, lavando as mãos, e deixando a juventude seguir adiante sem cultivar nenhum valor elevado de civilidade. E essa falha é culpa dos políticos que não colocam nas exigências curriculares a formação moral, ética, cultural, compostural, do caráter, da personalidade. Só querem saber de treinar os alunos para acertar as questões do ENEM. E com isso, "a vaca vai pro brejo" num descalabro colossal que nos levará, brevemente, ao alicerce do fundo do poço. Temos que botar a boca no trombone e denunciar essa vergonha nacional que nos coloca em papel ridículo no concerto das nações. Um país que, verdadeiramente, NÃO É SÉRIO. Eu recomendaria ao resto do mundo que não mantivesse nenhuma relação nem diplomática nem comercial com o Brasil por décadas. Até que todo esse descalabro, incluindo aí a corrupção dos políticos, empresários, magistrados, policiais, advogados, funcionários públicos, pastores eclesiásticos, padres, bispos, militares, e toda essa turba fosse colocada devidamente em presídios sem atenuação de pena, sem regalia nenhuma e com trabalhos forçados. Mas trabalhos braçais. Enquanto a porção de pessoas de bem do país não tomar atitudes rígidas contra essa malandragem toda, nada se resolverá. E isso não é nenhuma atitude direitista, como alguns dizem, por confundir esquerda com bagunça e licenciosidade. A verdadeira esquerda é ordeira, é disciplinada, é diligente, é muito séria, não é permissiva, não é devassa, não é leniente.

O capitalismo funciona, pois ele não é uma ideia , uma abstração (como o comunismo, socialismo,etc), é parte intrínseca da natureza humana , não precisa ser explicado, é instintivo. O que o mestre tem a dizer sobre esse frase? Concorda, discorda? X da questão.

Discordo. Porque o capitalismo não é a situação em que as pessoas trabalhem e amealhem capital e sim a situação em que algumas pessoas explorem o trabalho das outras para amealhar capital sem trabalhar. Isso não é natural. isso é criminoso. Um capitalista é um escravocrata. O capital é bom, o capitalismo é mau. No comunismo existe capital. Mas ele é distribuído por todo mundo. Capital é necessário e bom. Capitalismo não é necessário e é ruim. O capital não pode ficar restrito a poucas pessoas. Todos têm que ser detentores de capital e, assim, ninguém ser empregado de ninguém. Isso é que é a situação natural da humanidade. Não é natural que ninguém seja empregado de ninguém. Isso não é intrínseco da condição humana coisa nenhuma. Isso é o que pessoas que se valem da sua força e maior inteligência malévola impõem ao resto da população. É uma verdadeira escrevidão. totalmente ignóbil. Tem que ser algo expurgado da face da Terra de tão malvado que é.

O senhor poderia, por gentileza, me indicar livros para entender melhor de política e economia? Lembro do senhor falar que, quem estuda, sabe que o socialismo e o comunismo não funcionariam. Obrigada!

Nunca disse isso. O que eu sempre digo é que o socialismo de estado, acompanhado da ditadura do proletariado não funciona. Mas o comunismo anarquista não estatizante funciona sim. E é o melhor sistema econômico para a humanidade. A maioria dos livros disponíveis sobre economia é capitalista ou marxista, que são ambos péssimos. Vou procurar se acho alguns livros sobre economia não capitalista, não marxista e, principalmente, não monetária.

digo isso porque o senhor falou que a filosofia e a ciencia sao incompativeis e que a ciencia e mais importante que a filosofia, considero entao a sua nova resposta?

O que eu já disse é que Religião e Ciência são incompatíveis e não que Filosofia e Ciência sejam incompatíveis. Jamais poderia ter dito isso. Por favor, me passe o link em que eu tenha dito isso. O que eu dito muito é que vários filósofos são ignorantes em Ciências e, com isso, dizem besteiras. Ou que muitos cientistas são ignorantes em Filosofia e, com isso, também dizem besteiras.

mas se ha explicaçoes cientificas que um fisico estuda, por que é que ele tem que estudar filosofia na hora de pesquisar ou divulgar? a fisica nao e mais importante que a filosofia?

Não. A Filosofia é muito mais importante do que a Física. Sem sombra de dúvida. A Filosofia é a mestra da vida. É a Filosofia que nos capacita a saber refletir, a raciocinar, a ponderar, a ter sabedoria. Além disso, um Físico ou qualquer profissional, não tem que ser treinado para escrever artigos para publicar. Essa é uma concepção inteiramente idiota da profissão de Físico. É uma concepção carreirista. Um físico tem que entender como o mundo funciona. Isso é que é a Física. Então os físicos ficam pesquisando como as coisas funcionam e publicam artigos para que os outros tomem conhecimento do que ele descobriu e possam refutar ou dar prosseguimento. Não para colecionar artigos publicados e ganhar mais pontos em concursos. Quem faz assim é um físico mesquinho, que não interessa pelo progresso da ciência e sim pelo progresso da sua carreira. E para entender como a sua contribuição se insere no progresso global do conhecimento humano o físico tem que conhecer filosofia. Senão ele é meramente um técnico e não um cientista. Todos os grandes físicos se enveredaram pelos aspectos filosóficos de suas descobertas e promoveram grandes discussões filosóficas. Exceto alguns que têm ojeriza pela Filosofia, como o Hawking, que não é um físico tão grande assim.

Eu ouvi falar que o açúcar emburrece, só para estarmos em sincronia quanto ao assunto, que tipo açúcar está se referindo?

Estou falando da glicose, que é produzida pela sacarose, pela frutose, pelo amido e outros açúcares. A glicose é o principal combustível do cérebro. O problema é que as pessoas ingerem muito mais do que o necessário e o excesso prejudica. Esse muito mais costuma ser 400% a mais. Isso atrapalha a cognição. Mas o maior problema de falta de cognição nos desnutridos é a falta de açúcar.
http://www.lowcarb-paleo.com.br/2013/01/o-cerebro-nao-precisa-de-glicose-o.html
http://www.huffpostbrasil.com/2015/04/18/acucar-efeito-cerebro_n_7081806.html
https://pt.wikipedia.org/wiki/Neuroglicopenia
http://www.centrovegetariano.org/Article-225-Alimentando%2Bos%2Bneur%25F3nios.html

Autodidatas podem aprender tão ou melhor ainda do que aqueles que aprendem sob a supervisão de um professor? Há situações/pessoas em que seria preferível aprender sozinho? Eu diria que sim.

A vantagem de se aprender sozinho é que se anda bem mais rápido, porque, na turma, o professor tem que ficar tirando as dúvidas dos que não entendem. Todavia há a questão de aparecer uma dúvida e não ter a quem recorrer. O que eu sugiro, e sempre fiz, é ir estudando por conta própria dentro da turma e ir avançando bem além do que esteja sendo abordado, inclusive com o concurso de material em paralelo, que amplia bastante o conhecimento, não só na abrangência, mas também na profundidade. Quando se deparar com alguma dúvida, então pergunte ao professor. Todavia é preciso que se esteja sempre bem a par do que esteja sendo dado em classe, porque o professor pode querer dar uma rasteira em alunos "sabichões", perguntando o que está sendo dado na hora. Aí não pode haver vacilo. Tem que dar um show. Por sorte eu sempre agi assim e sempre me dei bem. Às vezes os professores é que tiravam dúvidas comigo. Mas eu tinha vários livros de cada assunto, desde o tempo em que estava no ensino fundamental. Porque os meus pais eram bibliófilos e pensavam dessa forma. A pessoa de um livro só sobre cada assunto é sempre medíocre.

Alonso & Finn vol I ou Moyses vol I para um estudante que não domina o cálculo II ainda? Pretendo estudar com rigor, mas alguns dizem que o Moyses exagera (aparentemente em especial no volume II) e outros dizem o mesmo do Finn. Já tenho materiais mais "leves" para ir nivelando paralelamente.

De fato o Alonso & Finn e o Nussenszveig são livros de Física Geral mais exigentes em Matemática, especialmente o Alonso & Finn. Se você tem esse tipo de problema é preferível estudar pelo Halliday, o Young ou o Knight. O Curso de Berkeley, que, conceitualmente é o melhor, também exige matemática mais avançada.

Uma pessoa pode ser sábia sem ser inteligente?

Pode. A sabedoria é a virtude de se achar a solução correta para alguma questão, especialmente ligada às disputas da vida. Ela não requer uma grande inteligência geral e, nem mesmo, uma grande sapiência. Mas requer alguma. A sabedoria é a arte de aconselhar a tomar as decisões mais acertadas, inclusive para si mesmo. Acertadas no sentido de promoverem a maximização da felicidade para o maior número de seres. Essas é que são as decisões sábias e as decisões justas. Sabedoria é saber identificar onde está o bem e escolhê-lo. É saber peneirar as aparências e encontrar o âmago de tudo. Aquilo que, de fato, é o que produzirá resultados. Uma pessoa mais inteligente e mais bem informada, com grande sapiência e cultura tem mais condições de ser mais sábia. Mas não necessariamente. Há pessoas muito inteligentes que não são muito sábias. Que fazem escolhas equivocadas para sua vida. Não me refiro a escolhas em termos de sucesso financeiro, obtenção de poder ou fama. Isso é balela. As escolhas que importam são as escolhas, por exemplo, do cônjuge, da profissão, da religião, se for ter alguma, do partido político, se for ter algum, das concepções ideológicas, do estilo de vida. Isso é que faz diferença para a felicidade da pessoa e dos outros. Para isso não é preciso que se seja muito inteligente. Mas é preciso que se seja inteligente. Todavia há o reverso da medalha, isto é, pessoas de extremada burrice que vivem felizes por total falta de consciência de suas próprias condições de vida. Mas eu não diria que ser assim é ser sábio.

o senhor acha que todo fisico deve aprender filosofia ou ela nao tem a menor importancia nas pesquisas que um fisico pode fazer?

Acho que é importantíssimo para todo físico ser, também, bem entendido em Filosofia. Não em história da Filosofia, mas que saiba filosofar e bem. Isso é imprescindível. Da mesma forma que todo filósofo tem que entender razoavelmente bem de Física e de Biologia. Senão vai dizer muita besteira, do mesmo modo que o físico que não entende de Filosofia.

Professor.. você que tem uma inteligência acima da média, o seu corpo necessita de mais nutrientes do que a maioria? Coisas bem específicas para as suas células nervosas.. e etc?

Em verdade precisa de açúcar, mas eu não como porque sou diabético. Em compensação como frutas, que também têm açúcar, menos concentrado, além de vitaminas e fibras. Como nozes, castanhas. Como folhas escuras. E, o importante, bebo muita água. Todavia não como carne. Mas como laticínios e ovos. Só que pouco ovo, pois não gosto muito. Além, é claro, de arroz, feijão, macarrão, pão, batata. Mas não muito. E chocolate (sem açúcar).

Professor, estava discutindo recentemente com um historiador da física em que ele argumentou que o teorema de Noether prova que a relatividade geral é falsa. Eu fiquei simplesmente espantado com essa afirmação. Ainda mais que ele disse que todos os físicos sabem disso, inclusive Einstein (continua) que sabia disso. Pois bem, ainda não me debrucei no tal teorema, estou no segundo ano de física e provavelmente estudarei esse ano em “mecânica clássica”. Mas o que você poderia dizer sobre isso? (continuação) que sabia disso. Pois bem, ainda não me debrucei no tal teorema, estou no segundo ano de física e provavelmente estudarei esse ano em “mecânica clássica”. Mas o que você poderia dizer sobre isso.

Fiz mestrado em Relatividade Geral e nunca ouvi dizer que existe essa prova. Vou buscar saber, mas não acho que seja verdade, muito menos que Einstein sabia disso.

Você tem buscado sua paz de espírito? Como?

Não. Minha mente vive normalmente em paz e o que falta de paz é o que eu quero de agitação para desenvolver meus projetos de melhoria do mundo.

Qual é a melhor forma de relaxar?

Dormir

Qual a sua opinião sobre a colocação de piercings em adolescentes? O adolescente decide? Os pais decidem? Devem ser colocados somente após a maioridade?

O adolescente decide, ora. Ele é que vai carregar o piercing. Mas deve combinar com os pais. Em caso de discordância, tem duas alternativas: ou atende a vontade dos pais e não põe o piercing e fica triste ou põe o piercing e rompe com a família, criando o constrangimento de ter que ir morar fora por conta própria ou continuar em casa de mal com a família pelo resto da vida. Pesando os prós e os contras, acho preferível ficar triste por não por o piercing.

https://ask.fm/wolfedler/answers/140510636061 Ernesto, isso é uma característica sua ou você acha que todo livre-pensador deveria ser assim? Por quê?

Para mim as grandes virtudes de um filósofo são ser um livre pensador e um cético. Sem isso não se é filósofo. Assim, considero que todo filósofo precisa ter a mente aberta para qualquer concepção e examinar tudo. Então estudar, comparar, concordar, discordar, refletir, escolher, inventar, propor e apresentar a sua concepção sobre o tema, sempre de modo provisório mas, enquanto não houver fortes razões para mudança, estar munido de fortes argumentos para defender seu ponto de vista, que, todavia, jamais pode ser cristalizado como algo imutável. Por isso não gosto de rótulos, isto é, de dizer que sou empirista, racionalista, realista, materialista, idealista, positivista, pragmatista, utilitarista, criticista, fisicalista, fenomenologista, analítico, existencialista, pós-modernista ou o que seja. Em verdade concordo e discordo de pontos específicos desta ou daquela corrente, podendo dizer que há algumas em que eu concordo com a maior parte e outras com quase nada. Assim eu acho que todo filósofo deva ser: "não adjetivado", apenas "filósofo". Muitos filósofos discordam desse meu ecletismo, considerando que, assim, eu não sou nada. Pois eu acho, justamente, é que, sendo nada, é que sou verdadeiramente filósofo. Além disso acho essencial que todo filósofo tenha um bom conhecimento de ciências, especialmente física e biologia, além da matemática, bem como um bom conhecimento de artes. Senão não vai saber interpretar o fazer humano, que é, justamente, o seu metier.

Por que qualquer atividade, mesmo as mais desagradáveis ou enfadonhas, se torna mais prazerosa se a fizermos ouvindo música? Seria porque prestamos atenção 'a música ou esta exerce algum efeito sobre o cérebro? E mais: acredita que a música atrapalha na concentração de quem estuda, por exemplo?

De fato a música provoca uma leveza de espírito que nos permite encarar as dificuldades com menor enfado e, com isso, despachá-las sem grade desânimo. É um efeito real sobre o cérebro, que propicia uma ligação com lembranças felizes de eventos agradáveis. Mas é preciso que essa música seja ligeira e aprazível. Do mesmo modo que a música que pode ser usada para acompanhar o estudo, inclusive com bom proveito. Não pode ter uma sonoridade muito intensa e nem ser entrecortada por mudanças abruptas de intensidade ou andamento. O ideal, para mim, é uma música de Mozart, de Bach, de Chopin, de Debussy. De preferência apenas instrumental, pois a letra pode prender a atenção, desviando-a do estudo. Não recomendo Scriabin, Shostakovich, Mahler, nem mesmo Beethoven para fundo musical de estudos.

Uma sala de aula deveria ter quantos alunos?

O que não deveria é existir sala de aula, Mas, uma vez que existe, o número de alunos depende do nível de ensino em questão. No infantil, uns dez, no primário, uns vinte, no fundamental, uns trinta, no médio, uns quarenta, no superior, nas disciplinas de massa pode ter uns duzentos, mas nas profissionalizantes dos últimos períodos, uns trinta. Na pós graduação, uns quinze. Isso é um palpite meu com base em minha experiência em ter lecionado por mais de quarenta anos nesses níveis todos (exceto o infantil e o primário), em turmas de varáveis números de alunos, com mais de vinte mil horas de aula para mais de quatro mil alunos.

Alguns economistas afirmam que o dinheiro contado em bilhões e trilhões de qualquer moeda não existe em espécie. Seria apenas um dinheiro em forma de dados virtuais. Corremos o risco de um colapso financeiro ou estamos caminhando para uma era em que o intangível substituirá o palpável?

De fato, esse dinheiro não existe em espécie palpável, seja em papel moeda, seja em lastro de qualquer espécie. É um dinheiro meramente contábil, um registro em contas correntes que nem são mais em livros, mas em bancos de dados de computadores, portanto virtual mesmo. Mas ele corresponde a lucros, pagamentos, rendimentos, recebimentos de valores correspondentes a transações reais, mesmo que, muitas vezes, valorizadas fiticiamente. De modo que essa substituição do palpável pelo intangível não é problema nenhum. Isso já foi até mais forte no tempo em que a palavra dada tinha o mesmo valor do que um contrato assinado.

Professor,quando Nietzsche afirmou que Deus está morto ele se refere a institucionalização do mesmo:igreja,bíblia,imagem,etc;pelo menos foi isso que eu entendi pelos livros.Mas o que seria ao certo um Deus para Nietzsche?

O que Nietzsche quis dizer, uma vez que, sendo ateu, não poderia considerar que algo não existente morresse, é que o "conceito de de Deus", que as pessoas têm na cabeça e que acreditam que corresponda a uma entidade verdadeiramente existente, teria morrido. Ou seja, que a humanidade teria concluído que Deus não existe e, portanto, deixaria de considerar a sua existência como real. Todavia ele se enganou fragorosamente. A humanidade, em sua maioria, considera que esse conceito corresponda a uma entidade verdadeiramente existente. Outro modo de entender a sua frase é que ela signifique que a ciência e a filosofia teriam, finalmente, conseguido mostrar, sem sombra de dúvida, que o conceito de Deus não corresponde a nenhuma realidade fática. Nisso ele pode estar certo.

Mestre, o que pensas sobre o mito de que homens negros têm os orgãos maiores do que os demais? E sobre a questão deles serem superiores no atletismo. Vide Usain Bolt. O que tens a dizer sobre, fato ou mito ?

Não conheço estatísticas sobre o tamanho dos pênis de humanos das diferentes raças. Somente um levantamento desse é que pode atestar qualquer conjectura. Sendo uma comprovação fática, não há que se contestar. Da mesma forma o desempenho no atletismo. Como esportes e atletismo não são assuntos do meu interesse, nunca procurei saber a respeito. Ademais, considero que qualquer que seja o resultado desses dois levantamentos, não faz diferença nenhuma. Que vantagem há em se pertencer a uma raça que, na média, tem pênis maiores ou que, na média, tem melhores desempenhos atléticos?

Você, jogou pedra na cruz?

Claro que não. Que ideia mais besta. Em relação ao cristianismo o que eu acho é que é uma ideologia equivocada, baseada em concepções falsas, como a da existência de Deus, da divindade de Jesus Cristo, do pecado original, da redenção de Jesus Cristo, da existência de uma alma imortal, da existência de céu e inferno, de anjos e demônios, no caso dos católicos, da presença real de Jesus na Hóstia Consagrada, de infalibilidade do Papa e várias outras totalmente descabidas. O que eu faço, porque considero meu dever de consciência, é esclarecer as pessoas cristãs do grande erro que cometem em o sê-lo. Mas nem por isso vou condená-las, inclusive porque há muitas que o são com sinceridade e piedade, não se configurando, em absoluto em pessoas "do mal". Mas há outras que são, como o Edir Macedo e seus assemelhados. Nesses eu desço o pau sem piedade. Quanto ao próprio Jesus Cristo, se existiu, até que eu admiro o que se diz que ele tenha dito e, mesmo, procuro seguir, tirando fora a parte sobrenatural. Penso que ele foi um coitado totalmente injustiçado pelos judeus e pelos romanos. Todavia sua mensagem primordial foi totalmente deturpada, começando por Saulo de Tarso e continuando por Constantino, quando oficializou o cristianismo. Nenhuma religião pode ser oficial de estado nenhum. Nem o judísmo em Israel, nem o islamismo nos países muçulmanos, nem o cristianismo na Inglaterra. Crença religiosa só pode ser livre, incluindo aí a liberdade de descrença em todas.

Professor, tenho você como meu mestre, meu guru, apesar de não concordar com todas as suas posições. O senhor se incomoda com isso?

Não, absolutamente. Sinto-me lisonjeado. Mas não considero que tenha esse estatuto de "guru", porque, em verdade, não sou um mestre. Sou um aprendiz. Sempre estou aprendendo e mudando minhas concepções. Não se pode dizer que tenho uma escola de pensamento. Sou um eclético e um eclético nunca é um mestre de nenhuma escola de pensamento ou científica. Para cada assunto tenho uma opinião diferente. Sou combatido por capitalistas e por comunistas, por teístas e por ateus, por libertários e por conservadores. Porque, ora concordo com o ponto de vista de um, ora com o do seu oposto. Mas gosto de levantar questões, de ser polêmico, de refletir sobre tudo, de concordar e discordar com veemência. De duvidar. Acho que faço um trabalho de relevo em despertar esse tipo de comportamento na juventude. Bem como de despertar a curiosidade sobre tudo.

Existem pessoas sem cultura?

Depende do que você está chamando de "cultura". No sentido mais amplo, cultura é o cabedal de saberes e práticas de um povo. Assim, como todos são pertencentes a algum povo, isto é, um conglomerado de pessoas com várias afinidades de costumes, hábitos, procedimentos, história, conhecimentos, então todos têm uma cultura a que pertencem e a assimilam como sua.
Outro conceito mais restrito de cultura já significa um cabedal de saberes, práticas, costumes, hábitos, procedimentos, modos de ser, modos de falar, específicos de um grupo mais refinado, com mais amplitude e profundidade dessas características todas. Por esse conceito há pessoas mais cultas e menos cultas, conforme o que saibam ou não desse universo de saberes e procedimentos ditos, então, "cultos".

Ernesto, o que você pensa sobre o fisiculturismo? Você acha que isso deveria deixar de existir?

Acho uma rematada bobeira. Mas não acho que deveria deixar de existir, porque ainda haverá gente boba no mundo. Se eles se comprazem com isso, não tem problema. Pode ser que eles, por sua vez, achem boba gente que aprecia assistir óperas, como eu. De modo que essa negócio de bobeira, depende da opinião de cada um. Há quem corte cabelo no estilo moicano e outros que o deixam despenteado "a la Einstein", como eu às vezes faço. Cada qual com a sua bobice.

Quais dicas, conselhos você concede para um adolescente que fará sexo pela primeira vez e pretende ter uma boa experiência?

Para começar que o faça com outra menina adolescente que também o fará pela primeira vez. Isso é fundamental, porque assim ambos estarão em igualdade de condições. Depois que se dispam não só das roupas mas de todo e qualquer pudor. Não se pode fazer sexo bom com pudor. E que experimentem tudo o que quiserem sem restrição a não ser a de forçar a fazer algo que o outro não queira. E que, por uns tempos, mantenham essa parceria fixa, justamente para progredirem no aprendizado. É válido, também, assistirem vídeos eróticos em que as relações sejam apresentadas explicitamente, mas de modo bonito e sensual, para que se tenha exemplos do modo bom de se fazer sexo.

Essas brigas são reflexo do país ou é só rivalidade? http://globoesporte.globo.com/rj/futebol/noticia/2017/02/por-determinacao-da-justica-classicos-no-rj-terao-torcida-unica.html

São reflexo de uma mentalidade mesquinha, tacanha, boçal, completamente mal educada, inteiramente despida de qualquer nobreza que grande parte do povo brasileiro, especialmente homens jovens, exibe, até com orgulho, justamente de ser truculento, violento, mal educado, desrespeitador, boçal e esse tipo de coisa. Isso tem que ser objeto de um programa muito amplo e prolongado (séculos a fio) nas escolas, especialmente do ensino fundamental I, para mostrar aos meninos o quanto é deplorável, estúpido, feio, deselegante e tudo de ruim ter esse tipo de comportamento. Antes de mais nada é preciso convencer que o bom é ser elegante e não deselegante, é ser gentil e não estúpido, é ser bem educado e não mal educado. E que ser assim não é boiolice nenhuma. Não é fácil mudar essa mentalidade, porque ela está enraizada, até mesmo, nas mães. E é muito difícil para a escola ensinar os alunos a contestarem os pontos de vista dos pais. Mas tem que ser feito por meio de um programa governamental de duração plurisecular.

Ernesto, você é parente de Friedrich Rückert, o poeta?

Sim. Há muitas gerações, pois ele foi do século 18. Meus parentes austríacos do século 19 eram professores universitários. E adquiriram um título de nobreza. O "von" no sobrenome. Só que meu avô era republicano e a favor da libertação da Hungria e da Tchecoslováquia do domínio da Áustria. Meu bisavô fora professor de Química do Arquiduque Francisco Ferdinando, assassinado em Serajevo. Meu avô era professor de Russo e Tcheco. Mas também falava, além do Alemão, Inglês, Francês, Italiano, Espanhol e Português (de Portugual). De modo que ao vir para o Brasil, em 1906, para fugir da prisão e possível condenação à morte por alta traição, acabou se casando com a filha do adido comercial do Consulado Português em Belo Horizonte.

O que vc acha dessa medida de proibirem torcida visitante nos clássicos de futebol de SP,RJ? Pra vc, essa ideia de clássico com torcida única é eficaz? Só lembrando que a maioria das brigas entre torcidas ocorre na rua... CONTINUA

Acho o cúmulo da demonstração de incivilidade do povo. Tem que deixar todo mundo junto e, melhor ainda, misturado. Com homens, mulheres e crianças. Quem promover algum tumulto, então, tem que ser preso e deixado na cadeia por uns bons sete anos com a perda de todos os direitos, de todo o salário, de qualquer regalia, de qualquer indulto, de qualquer visita, até que se suicide.

Podemos falar de justiça, mas se nada fizermos para proteger nosso direito, e ficarmos somente com nosso individualismo; sempre haverá pessoas de mal caráter no poder, de prontidão para violar os direitos do povo em benefício próprio. O mestre concorda com essa frase?

Não. O progresso civilizatório da humanidade levará à total extinção de qualquer pessoa de mal caráter no mundo. Isso é uma questão de educação que em alguns séculos ou poucos milênios será resolvida.

Ernesto, quais livros de física você usou para aprender física? Como você tomava nota desses livros?

Quando eu estava no científico o livro adotado era o Antônio de Souza Teixeira Júnior que, depois, foi mudado para o Dalton Gonçalves. Nenhum dos dois, todavia, eu considerei como bons. Na faculdade de matemática foi adotado o Halliday-Resnick, na edição antiga em dois volumes grossões, um de capa vermelha outro de capa azul, editados pela Livros Técnicos e Científicos. Esses livros todos me davam o traquejo necessário para resolver com facilidade qualquer problema de Física. Mas eu só vim a aprender Física mesmo, pra valer, com a compreensão cabal dos fenômenos que aconteciam (e isso é que é o mais importante, não o traquejo para resolver problemas, que também é importante, mas secundário), quando eu comecei a lecionar na Escola Preparatória de Cadetes do Ar, de Barbacena, que adotava a coleção do Physical Science Study Committee (PSSC), não só com o livro texto, mas com todo o material complementar: a coleção de 52 filmes, as cerca de 60 práticas de laboratório com kits replicados 20 vezes para que pudessem todas serem feitas em duplas de alunos e, especialmente, com o "Guia do Professor", que não era apenas um livro texto com respostas dos exercícios e problemas mas um outro livro, com diretrizes sobre o melhor modo de abordar e explicar cada assunto. Como eu lecionei para as três séries do científico, pude me inteirar do conteúdo todo. Ainda mais que, no segundo semestre da terceira série, a EPCAR adotava o complemento dos "Tópicos Avançados" (traduzido por nós mesmos - https://pt.scribd.com/collections/17891824/PSSC-Topicos-Avancados) que abordava tópicos de Física Moderna (relatividade, quântica, atômica, nuclear, estado sólido, estatística, momento angular). Em razão disso eu resolvi adquirir o "Curso de Física de Berkeley" que é a continuação do PSSC em nível superior. O melhor curso de Física Geral para universidades. Outro muito bom que eu também comprei nessa época (1968) foi o "Alonso e Finn". Esses são os livros que recomendo. Realmente compensa investir em sua compra. O Alonso e Finn tem uma edição atual de Portugal. O Berkeley e o PSSC só são encontrados em sebos. Os dois primeiros volumes do Berkeley foram traduzidos para o português. Os três últimos só existem em Inglês e Espanhol (de editora Reverté). Livros atuais de Física para o nível médio são todos ruins porque só pensam em treinar os alunos para resolver questões do ENEM e de Vestibulares e não em aprender Física pra valer. Uma lástima.

O que você acha dessa frase: "Jamais diria te amo para um filho meu homem."?

Totalmente besta, quer seja por parte do pai ou da mãe. Ama-se os filhos tanto masculinos quanto femininos e isso tem que ser dito para eles sim. Inclusive para que fique bem claro, não apenas por atitudes, que eles contam com esse amor, para sua estabilidade psíquica. E isso pode ser dito a qualquer idade e não apenas na infância. Tenho um filho de mais de trinta anos e digo que o amo, como amo minha filha. Isso não significa que eu o ame eroticamente, de modo nenhum. Quem pensa desse jeito é uma pessoa de mentalidade totalmente distorcida e mesquinha.

https://ask.fm/wolfedler/answers/140484018205 1) Mas essa visão bem x mal é maniqueista. Isso depende do ponto de vista. Desconheço alguma pessoa perfeita nisso. 2) Nem todo ateu é engajado em algo. Assim como ateismo não pressupõe imoralidade tambem não pressupõe moral. Isso é de cada pessoa.

Não me referi à noção maniqueísta de bem e de mal como entidades opositoras. Todavia a noção de bem e de mal existe mesmo, com ou sem maniqueísmo (e eu considero que seja sem maniqueísmo). Ateus, por serem ateus não são bons nem maus. Da mesma forma que não possuem nem deixam de possuir uma moral. O que eu rejeito é a concepção de que alguém, por ser ateu, não tenha moral e que por isso, seja mau. Bons e maus os há entre ateus e crentes. Geralmente uma pessoa boa tem uma moral, mas nem sempre a sua moral é uma moral eticamente válida. Católicos que até foram canonizados santos tinham a moral de que seria certo queimar hereges. Mesmo sendo seguidores de Jesus Cristo, que era um herege. Outra questão é que niilismo e ateísmo são concepções totalmente independentes. Ateu não precisa ser niilista.

Acha que carisma de pessoas como Ariano Suassuna pode causar deseducação como fez aqui https://youtu.be/TgP2tRVySX0 ? E pra isso se precisra de mais refutações como essa https://youtu.be/FMUhEN_i6S0

Infelizmente, sim. Ele está revelando uma profunda ignorância científica, um profundo preconceito contra a ciência e isso pode ser extremamente danoso para a juventude. A refutação do Pirula está excelente como, aliás, a maior parte de todos os seus vídeos, que eu aprecio muito. Faz falta que as escolas, não apenas ensinem a evolução em suas aulas de Biologia, mas que também mostrem como as concepções criacionistas são totalmente fajutas. Tem que derrubar essas idéias erradas mesmo. Não importa que isso vá contra as concepções religiosas das pessoas. Essas concepções estão ERRADAS e isso tem que ser mostrado de modo claro e inteiramente convincente.

https://ask.fm/wolfedler/answers/140493894173 A palavra de "espirito" pode ser figura de linguagem e não literal.

Certamente. Mas do que eu conheço do Samadhi o espírito a que ele se refere é, justamente, a alma, a entidade sobrenatural que comandaria a nossa mente. Claro que se pode fazer uso da palavra "espírito" com outra conotação e eu mesmo faço esse uso. Com a conotação do aspecto da mente mais voltado para considerações superiores, para a virtude, para o bem. Sem considerar que a mente possua qualquer ingrediente sobrenatural. Estar com o "espírito elevado", ou o "espírito tranquilo", então significaria estar com o pensamento colocado em temas nobres ou estar com a mente completamente desprovida de qualquer perturbação, aflição ou agitação.

Se a tranquilidade da água já permite-nos refletir as coisas, o que a tranquilidade do espírito não poderá? – Chuang Tzu

Bela frase, mas completamente sem sentido, pois espírito não existe e água sim.

por que alguns fisicos tem preconceito contra os engenheiros, muitas vezes menosprezando eles? A matematica e fisica que um engenheiro geralmente sabe é inferior a de um fisico?

Isso é uma grande bobeira dos físicos. Depende do ramo da Física ou da Engenharia o fato da Matemática envolvida ser mais ou menos complicada. Há muitas coisas que um engenheiro sabe fazer e que um físico não sabe e vice-versa. Por exemplo, projetar um foguete que vá pousar num ponto determinado de certa Lua de Saturno é uma proeza de engenharia que poucos físicos são capazes de fazer. Geralmente os físico se referem aos engenheiros roskoff, que só sabem fazer o que já está previsto e completamente tabelados nos manuais. Esses são engenheiros medíocres mesmo. Mas podem haver físicos medíocres também. Uns e outros são os que não inventam nada, não descobrem nada de novo, só sabem fazer o que os outros descobriram.

Professor Ernesto, o que acha dos misantropos? Já se considerou um? Talvez age como um diante de determinadas pessoas?

Nem de longe já fui, sou ou penso que poderia vir a ser um misantropo. O que não significa que não preze meus momentos de recolhimento solitário, nos quais me dedico, principalmente, a pensar, mas também a estudar, ler, ouvir música, assistir filmes, sem a companhia de ninguém. Note que não sou uma pessoa que sinta falta de sempre estar acompanhado, mas que gosto de estar acompanhado, que gosto de conversar, de namorar e de companhia. Só que não ininterrupta. O verdadeiro misantropo não gosta da companhia de outras pessoas. Isso é uma patologia, o que não é o caso da personalidade introspectiva. Em verdade não posso dizer que sou introspectivo, pois há momentos em que sou bastante extrovertido. Não ajo como misantropo diante de ninguém. Sou uma pessoa completamente sem vergonha. Falo tranquilamente com qualquer um sem o menor receio. Pode até ser o Papa ou a Rainha da Inglaterra. Ou o Torquemada de Santa Maria. Tenho uma elevadíssima auto-confiança. Acho que os misantropos precisam se submeter a um tratamento psicológico para superarem esse problema.

Vocês acham que o ask pode se tornar um lugar perigoso para pessoas de baixíssima auto-estima e emocional instável, visto que há muitas pessoas que gostam de atacar os outros em anonimo?

Pode. Mas é um risco que se tem que correr em razão dos grandes benefícios que ele é capaz de propiciar. O que é preciso é que as famílias e as escolas eduquem todas as crianças para que tenham grande auto-estima e grande estabilidade emocional, sendo capazes de enfrentar com sobranceria qualquer ataque maldoso de quem quer que seja.

Se os ateus niilistas são amoralistas, como eles podem criticar a imoralidade da religião?

O niilismo não é uma característica necessária do ateísmo. Aliás, a maioria dos ateus não é niilista. E alguém pode ser niilista sem ser ateu. O niilismo é uma concepção vazia e completamente infecunda, além de se revelar contraproducente para o encaminhamento da vida. E o pior. É uma concepção totalmente equivocada, isto é, falsa. errada. O ateísmo genuíno é completamente ético e considera que tanto o bem quanto o mal são existentes e que o bem tem total primazia sobre o mal, devendo cada pessoa pautar sua vida pela disseminação do bem e a erradicação do mal. Assim pensando é que os ateus consideram que as religiões são um mal e não porque não achem, como os niilistas, que não existe valorização ética (e nem estética e nenhuma outra) a respeito do que quer que seja. Consideram as religiões (todas) um mal, primeiramente porque são inverdades. E a verdade é um bem da máxima superioridade. Qualquer que seja a religião ela faz afirmativas errôneas a respeito da realidade do mundo, como, por exemplo, ao afirmar a realidade da existência de uma realidade sobrenatural. Depois as religiões, de modo geral, se ocupam de nortear o comportamento pessoal no sentido de garantir a salvação eterna, ou a abreviação do número de reencarnações, das pretensas almas das pessoas, por meio de uma série de rituais e práticas, características de cada uma, que, uma vez cumpridas, levariam ao objetivo de alcançar a salvação. Ora, não havendo nenhuma alma e nenhuma vida após a morte, nada disso tem o menor significado. Portanto fazer as pessoas seguirem tais práticas é um verdadeiro suplício a ser infringido a elas, sem o menor efeito, do mesmo modo que a prática de orações. O que os ateus pregam é que, para que o bem prevaleça sobre a Terra, todos devem agir de forma a disseminá-lo por meio da justiça, da honestidade, da fraternidade, da caridade, da solidariedade, da cooperação, da compaixão, da nobreza de atitudes, da bravura dos atos, da coragem, do altruísmo e da prática de todas as virtudes, com o que o mundo todo se tornará um lugar harmônico, fraterno, equânime, livre, aprazível, próspero e feliz para todos os seus habitantes.

LinkWithin

Related Posts with Thumbnails